Difference between revisions of "2005 AMC 10A Problems/Problem 10"

m
m (Solution)
Line 6: Line 6:
 
==Solution==
 
==Solution==
 
A [[quadratic equation]] has exactly one [[root]] if and only if it is a [[perfect square]].  So set
 
A [[quadratic equation]] has exactly one [[root]] if and only if it is a [[perfect square]].  So set
 +
 
<math>4x^2 + ax + 8x + 9 = (mx + n)^2</math>
 
<math>4x^2 + ax + 8x + 9 = (mx + n)^2</math>
 +
 
<math>4x^2 + ax + 8x + 9 = m^2x^2 + 2mnx + n^2</math>
 
<math>4x^2 + ax + 8x + 9 = m^2x^2 + 2mnx + n^2</math>
 +
 
Two [[polynomial]]s are equal only if their [[coefficient]]s are equal, so we must have
 
Two [[polynomial]]s are equal only if their [[coefficient]]s are equal, so we must have
 +
 
<math>m^2 = 4, n^2 = 9</math>
 
<math>m^2 = 4, n^2 = 9</math>
 +
 
<math>m = \pm 2, n = \pm 3</math>
 
<math>m = \pm 2, n = \pm 3</math>
 +
 
<math>a + 8= 2mn = \pm 2\cdot 2\cdot 3 = \pm 12</math>
 
<math>a + 8= 2mn = \pm 2\cdot 2\cdot 3 = \pm 12</math>
 +
 
<math>a = 4</math> or <math>a = -20</math>.
 
<math>a = 4</math> or <math>a = -20</math>.
  
 
So the desired sum is <math> (4)+(-20)=-16 \Longrightarrow \mathrm{(A)} </math>
 
So the desired sum is <math> (4)+(-20)=-16 \Longrightarrow \mathrm{(A)} </math>
+
 
 
==See Also==
 
==See Also==
 
*[[2005 AMC 10A Problems]]
 
*[[2005 AMC 10A Problems]]

Revision as of 10:49, 2 August 2006

Problem

There are two values of $a$ for which the equation $4x^2 + ax + 8x + 9 = 0$ has only one solution for $x$. What is the sum of those values of $a$?

$\mathrm{(A) \ } -16\qquad \mathrm{(B) \ } -8\qquad \mathrm{(C) \ } 0\qquad \mathrm{(D) \ } 8\qquad \mathrm{(E) \ } 20$

Solution

A quadratic equation has exactly one root if and only if it is a perfect square. So set

$4x^2 + ax + 8x + 9 = (mx + n)^2$

$4x^2 + ax + 8x + 9 = m^2x^2 + 2mnx + n^2$

Two polynomials are equal only if their coefficients are equal, so we must have

$m^2 = 4, n^2 = 9$

$m = \pm 2, n = \pm 3$

$a + 8= 2mn = \pm 2\cdot 2\cdot 3 = \pm 12$

$a = 4$ or $a = -20$.

So the desired sum is $(4)+(-20)=-16 \Longrightarrow \mathrm{(A)}$

See Also